ЗАДАЧИ
problems.ru
О проекте | Об авторах | Справочник
Каталог по темам | по источникам |
К задаче N

Проект МЦНМО
при участии
школы 57
Задача 78800
Темы:    [ Упорядочивание по возрастанию (убыванию) ]
[ Линейные неравенства и системы неравенств ]
Сложность: 4
Классы: 11
В корзину
Прислать комментарий

Условие

Даны два набора чисел: a1, ..., an и b1, ..., bn. Расположим числа ak в возрастающем порядке, а числа bk – в убывающем порядке. Получатся наборы
A1 ≤ ... ≤ AnB1 ≥ ... ≥ Bn.  Доказать, что  max{a1 + b1, ..., an + bn} ≥ max{A1 + B1, ..., An + Bn}.


Решение

  Перенумеруем числа в исходных наборах {ai}, {bi} так, что  bi = Bi  при всех i. Пусть для некоторых индексов  k < l  выполнено неравенство  al < ak.  Докажем, что если мы поменяем местами ak и al, то число   {ak + bk}  не возрастёт. Действительно,  ak + bk ≥ ak + bl  и   ak + bk ≥ al + bk.
  С другой стороны ясно, что такими операциями можно из произвольной перестановки чисел a1, ..., an можно получить перестановку A1, ..., An. Следовательно, max{a1 + b1, ..., an + bn} ≥ max{A1 + B1, ..., An + Bn}.

Источники и прецеденты использования

олимпиада
Название Московская математическая олимпиада
год
Номер 34
Год 1971
вариант
Класс 10
Тур 2
задача
Номер 2

© 2004-... МЦНМО (о копирайте)
Пишите нам

Проект осуществляется при поддержке Департамента образования г.Москвы и ФЦП "Кадры" .